-光と光学に関連する用語の解説サイト-

三角関数の最大値・最小値

本項では、『三角関数の最大値・最小値』の問題と解法について解説します。

本項では、以下のような最大値・最小値の問題の解き方について解説します。

問題(1)~(3)は、一次の三角関数の基礎的な問題です。

【基礎問題】
次の関数の最大値・最小値を求めよ。
\begin{eqnarray} \large (1)\hspace{5pt}y&\large =&\large \sin \left(\theta-\frac{\pi}{4}\right)\hspace{5pt}(0 \leqq \theta < \pi) \\[0.7em] \large (2)\hspace{5pt}y&\large =&\large \cos \left(\theta+\frac{\pi}{3}\right)\hspace{5pt}(0 \leqq \theta < \pi) \\[0.7em] \large (3)\hspace{5pt}y&\large =&\large \tan \left(2\theta-\frac{\pi}{3}\right)\hspace{5pt}(0 \leqq \theta \leqq \frac{\pi}{4}) \\[0.7em] \end{eqnarray}

(解答と解説 : 問題(1) 問題(2) 問題(3))

問題(4)~(7)は、三角関数の置換や、三角関数の合成などを使用する応用的な問題です。

【応用問題】
次の関数の最大値・最小値を求めよ。
\begin{eqnarray} \large (4)\hspace{5pt}y&\large =&\large -2\cos^2 \theta + 2\sin \theta +3\hspace{5pt}\\[0.2em] &&\large\hspace{40pt}(0 \leqq \theta < 2\pi) \\[1.2em] \large (5)\hspace{5pt}y&\large =&\large \tan^2 \theta -2\tan\theta +3\hspace{5pt}\\[0.2em] &&\large\hspace{40pt}(-\frac{\pi}{2} < \theta < \frac{\pi}{2}) \\[0.5em] \large (6)\hspace{5pt}y&\large =&\large \sin \theta - \sqrt{3}\cos\theta\hspace{20pt}\\[0.2em] &&\large\hspace{40pt}(0 \leqq \theta < 2\pi) \\[1.2em] \large (7)\hspace{5pt}y&\large =&\large\cos^2 \theta + 2\sin\theta \cos\theta -\sin^2 \theta\hspace{2pt}\\ &&\large\hspace{40pt} (0 \leqq \theta \leqq \frac{\pi}{2} ) \\[0.9em] \end{eqnarray}

(解答と解説 : 問題(4) 問題(5) 問題(6) 問題(7))

【1】最大値・最小値の基礎的な問題

本章では、基礎的な問題について解説します。

以下のような一次の三角関数の最大値・最小値を求める場合は、\(\displaystyle\large{\left(\theta-\frac{\pi}{4}\right)=t}\) など変数に置換すると解きやすくなります

【基礎問題】
次の関数の最大値・最小値を求めよ。
\begin{eqnarray} \large (1)\hspace{5pt}y&\large =&\large \sin \left(\theta-\frac{\pi}{4}\right)\hspace{5pt}(0 \leqq \theta < \pi) \\[0.7em] \large (2)\hspace{5pt}y&\large =&\large \cos \left(\theta+\frac{\pi}{3}\right)\hspace{5pt}(0 \leqq \theta < \pi) \\[0.7em] \large (3)\hspace{5pt}y&\large =&\large \tan \left(2\theta-\frac{\pi}{3}\right)\hspace{5pt}(0 \leqq \theta \leqq \frac{\pi}{4}) \\[0.7em] \end{eqnarray}

(解答と解説 : 問題(1) 問題(2) 問題(3))

【1-1】sinの問題

問題(1)
\(\large{0 \leqq \theta < \pi }\) のとき、次の関数の最大値・最小値を求めよ。
\(\displaystyle \large{y=\sin \left(\theta-\frac{\pi}{4}\right) }\)

【解答と解説】
問題の三角関数は、\(\large{\sin \theta}\) から \(\displaystyle\large{\frac{\pi}{4}}\) だけ平行移動した関数になっています。
この場合、\(\displaystyle\large{\theta-\frac{\pi}{4}}\) を別の変数に置き換えると解きやすくなります。

ここで、\(\displaystyle\large{\theta-\frac{\pi}{4} = t}\) とおきます。このとき、変数 \(\large{t}\) の範囲は \(\large{0 \leqq \theta < \pi }\) より、 $$\displaystyle\large{-\frac{\pi}{4} \leqq \theta -\frac{\pi}{4} < \pi-\frac{\pi}{4}}$$ $$\displaystyle\large{-\frac{\pi}{4} \leqq t < \frac{3}{4}\pi}$$ となります。

問題の関数を変数\(\large{t}\) で表すと、 $$\displaystyle\large{y=\sin t}$$ となります。ここで、\(\displaystyle\large{-\frac{\pi}{4} \leqq t < \frac{3}{4}\pi}\) を満たす範囲を単位円上に描くと以下のようになります。 三角関数の最大値・最小値の求め方_sinの計算

図中の単位円上の青線が \(\displaystyle\large{-\frac{\pi}{4} \leqq t < \frac{3}{4}\pi}\) を満たす範囲を表します。

この範囲内で \(\large{\sin t}\) は、\(\displaystyle\large{t=\frac{\pi}{2}}\) のとき最大値 \(\large{1}\)、\(\displaystyle\large{t=-\frac{\pi}{4}}\) のとき最小値 \(\displaystyle\large{-\frac{\sqrt{2}}{2}}\) をとります。

ここで、\(\displaystyle\large{\theta-\frac{\pi}{4} = t}\) から \(\large{\theta}\) の値に変換すると、
\(\displaystyle\large{\theta=\frac{3}{4}\pi}\) のとき最大値 \(\large{1}\)、\(\displaystyle\large{\theta=0}\) のとき最小値 \(\displaystyle\large{-\frac{\sqrt{2}}{2}}\)
となります。

【1-2】cosの問題

問題(2)
\(\large{0 \leqq \theta < \pi }\) のとき、次の関数の最大値・最小値を求めよ。
\(\displaystyle \large{y=\cos \left(\theta+\frac{\pi}{3}\right) }\)

【解答と解説】
問題の三角関数は、\(\large{\cos \theta}\) から \(\displaystyle\large{-\frac{\pi}{3}}\) だけ平行移動した関数になっています。
この場合、\(\displaystyle\large{\theta+\frac{\pi}{3} = t}\) と置き換えて考えます。

このとき、変数 \(\large{t}\) の範囲は \(\large{0 \leqq \theta < \pi }\) より、 $$\displaystyle\large{\frac{\pi}{3} \leqq \theta +\frac{\pi}{3} < \pi+\frac{\pi}{3}}$$ $$\displaystyle\large{\frac{\pi}{3} \leqq t < \frac{4}{3}\pi}$$ となります。

問題の関数を変数\(\large{t}\) で表すと、 $$\displaystyle\large{y=\cos t}$$ となります。ここで、\(\displaystyle\large{\frac{\pi}{3} \leqq t < \frac{4}{3}\pi}\) を満たす範囲を単位円上に描くと以下のようになります。 三角関数の最大値・最小値の求め方_cosの計算

図中の単位円上の青線が \(\displaystyle\large{\frac{\pi}{3} \leqq t < \frac{4}{3}\pi}\) を満たす範囲を表します。

この範囲内で \(\large{\cos t}\) は、\(\displaystyle\large{t=\frac{\pi}{3}}\) のとき最大値 \(\displaystyle\large{\frac{1}{2}}\)、\(\displaystyle\large{t=\pi}\) のとき最小値 \(\displaystyle\large{-1}\) をとります。

ここで、\(\displaystyle\large{\theta+\frac{\pi}{3} = t}\) から\(\large{\theta}\) の値に変換すると、
\(\displaystyle\large{\theta=0}\) のとき最大値 \(\displaystyle\large{\frac{1}{2}}\)、\(\displaystyle\large{\theta=\frac{2}{3}\pi}\) のとき最小値 \(\displaystyle\large{-1}\)
となります。

【1-3】tanの問題

問題(3)
\(\displaystyle\large{0 \leqq \theta \leqq \frac{\pi}{4} }\) のとき、次の関数の最大値・最小値を求めよ。
\(\displaystyle \large{y=\tan \left(2\theta-\frac{\pi}{3}\right) }\)

【解答と解説】
問題の関数の変数を \(\displaystyle\large{2\theta-\frac{\pi}{3} = t}\) と置き換えて考えます。

このとき、変数 \(\large{t}\) の範囲は \(\displaystyle\large{0 \leqq \theta \leqq \frac{\pi}{4} }\) より、 $$\displaystyle\large{0 \leqq 2\theta \leqq \frac{\pi}{2}}$$ $$\displaystyle\large{-\frac{\pi}{3} \leqq 2\theta -\frac{\pi}{3} \leqq \frac{\pi}{2}-\frac{\pi}{3}}$$ $$\displaystyle\large{-\frac{\pi}{3} \leqq t \leqq \frac{\pi}{6}}$$ となります。

与えられた関数を変数\(\large{t}\) で表すと、\(\large{y=\tan t}\) となります。
下図に \(\large{y=\tan t}\) のグラフを示します。図中の赤線は \(\displaystyle \large{-\frac{\pi}{3} \leqq t \leqq \frac{\pi}{6}}\) の範囲を表します。 三角関数の最大値・最小値の求め方_tanの計算

\(\large{y=\tan t}\) は \(\displaystyle\large{-\frac{\pi}{2} < t < \frac{\pi}{2}}\) において単調に増加する関数( \(\large{t}\) が増加すると、\(\large{\tan t}\) も増加する関数) であるため、\(\displaystyle\large{t=\frac{\pi}{6}}\) のとき最大値 \(\displaystyle\large{\frac{1}{\sqrt{3}}}\)、\(\displaystyle\large{t=-\frac{\pi}{3}}\) のとき最小値 \(\large{-\sqrt{3}}\) をとります。

ここで、\(\displaystyle\large{2\theta-\frac{\pi}{3} = t}\) から \(\large{\theta}\) の値に変換すると、
\(\displaystyle\large{\theta=\frac{\pi}{4}}\) のとき最大値 \(\displaystyle\large{\frac{1}{\sqrt{3}}}\)、\(\large{\theta=0}\) のとき最小値 \(\large{-\sqrt{3}}\)
となります。

【2】最大値・最小値の応用問題

次に 三角関数の置換や、三角関数の合成などを使用する応用的な問題について解説します。

【応用問題】
次の関数の最大値・最小値を求めよ。
\begin{eqnarray} \large (4)\hspace{5pt}y&\large =&\large -2\cos^2 \theta + 2\sin \theta +3\hspace{5pt}\\[0.2em] &&\large\hspace{40pt}(0 \leqq \theta < 2\pi) \\[1.2em] \large (5)\hspace{5pt}y&\large =&\large \tan^2 \theta -2\tan\theta +3\hspace{5pt}\\[0.2em] &&\large\hspace{40pt}(-\frac{\pi}{2} < \theta < \frac{\pi}{2}) \\[0.5em] \large (6)\hspace{5pt}y&\large =&\large \sin \theta - \sqrt{3}\cos\theta\hspace{20pt}\\[0.2em] &&\large\hspace{40pt}(0 \leqq \theta < 2\pi) \\[1.2em] \large (7)\hspace{5pt}y&\large =&\large\cos^2 \theta + 2\sin\theta \cos\theta -\sin^2 \theta\hspace{2pt}\\ &&\large\hspace{40pt} (0 \leqq \theta \leqq \frac{\pi}{2} ) \\[0.9em] \end{eqnarray}

問題(4),(5)は二次の三角関数が含まれているため、\(\large{\sin \theta = t}\) などと置換し、二次関数の最大値・最小値の問題として解く問題です。

また、問題(6),(7)は、三角関数の合成二倍角の公式 を利用して、関数を変形して解く問題です。

(解答と解説 : 問題(4) 問題(5) 問題(6) 問題(7))

問題(4) 置換により二次関数に変換する問題

以下の三角関数の最大値・最小値の問題の解き方について解説します。

問題(4)
\(\large{0 \leqq \theta < 2\pi }\) のとき、次の関数の最大値・最小値を求めよ。
\(\displaystyle \large{y=-2\cos^2 \theta + 2\sin \theta +3 }\)

【解答と解説】
問題の方程式は、\(\large{\sin}\) と \(\large{\cos}\) がどちらも含まれており、以下の相互関係の式を利用して変換します。 $$\large{\sin^2\theta + \cos^2\theta =1}$$ 上式から、与えられた関数を変形すると、 $$\large{y=-2(1-\sin^2\theta) + 2\sin \theta +3 }$$ $$\large{y=2\sin^2\theta + 2\sin \theta +1 }$$

ここで、\(\large{\sin\theta = x}\) に置換します。

このとき、\(\large{\theta}\) の範囲が \(\large{0 \leqq \theta < 2\pi }\) であるため、\(\large{x}\) の定義域は \(\large{-1 \leqq x \leqq 1 }\) となります。

問題の関数を 変数\(\large{x}\) によって置き換えると $$\large{y=2x^2+ 2x +1}$$ となります。上式から、三角関数の最大値・最小値の問題を 二次関数の最大値・最小値 の問題として解くことができます。

上記の二次関数を平方完成すると、 $$\large{y=2\left(x+\frac{1}{2}\right)^2+\frac{1}{2}}$$

ここで、上式から二次関数のグラフを書くと以下のようになります。 二次の三角関数を含む式を置換して二次関数を描いた図(1)

上図から、\(\large{x=1}\) のとき最大値 \(\large{5}\)、\(\displaystyle \large{x=-\frac{1}{2}}\) のとき最小値 \(\displaystyle \large{\frac{1}{2}}\) となります。

ここで、\(\large{\sin\theta = x}\) であるため、三角方程式 を解いて最大値・最小値となる \(\large{\theta}\) を求めます。
\(\large{\theta}\) の範囲が \(\large{0 \leqq \theta < 2\pi }\) であるとき、\(\large{\sin\theta = 1}\) のとき、\(\displaystyle\large{\theta = \frac{\pi}{2}}\) となります。

また、\(\displaystyle\large{\sin\theta = -\frac{1}{2}}\) のとき、\(\displaystyle\large{\theta = \frac{7}{6}\pi,\hspace{2pt}\frac{11}{6}\pi}\) となります。

以上から、
\(\displaystyle\large{\theta = \frac{\pi}{2}}\) のとき、最大値 \(\large{5}\)
\(\displaystyle\large{\theta = \frac{7}{6}\pi,\hspace{2pt}\frac{11}{6}\pi}\) のとき、最小値 \(\displaystyle \large{\frac{1}{2}}\)
となります。

問題(5) 置換により二次関数に変換する問題

以下の三角関数の最大値・最小値の問題の解き方について解説します。

問題(5)
\(\displaystyle \large{-\frac{\pi}{2} < \theta < \frac{\pi}{2} }\) のとき、次の関数の最大値・最小値を求めよ。
\(\displaystyle \large{y= \tan^2 \theta -2\tan\theta +3}\)

【解答と解説】
問題の関数は、二次の \(\large{\tan \theta}\) が含まれているため、\(\large{\tan \theta = x}\) とおいて解きます。

このとき、\(\large{\theta}\) の範囲が \(\displaystyle \large{-\frac{\pi}{2} < \theta < \frac{\pi}{2} }\) であるため、\(\large{x (=\tan \theta)}\) は全ての実数をとります。

問題の関数を \(\large{x}\) で表すと、 $$\large{y=x^2-2x+3}$$ となります。この二次関数を平方完成すると、 $$\large{y=\left(x-1\right)^2+2}$$ となります。

ここで、上式から二次関数のグラフを書くと以下のようになります。 二次の三角関数を含む式を置換して二次関数を描いた図(2)

上図から、\(\large{x=1}\) のとき最小値 \(\displaystyle \large{2}\) となります。また、\(\large{x}\) が全ての実数をとるため 最大値はありません。

ここで、\(\large{\tan\theta = x}\) であるため、三角方程式 を解いて最大値・最小値となる \(\large{\theta}\) を求めます。
\(\large{\theta}\) の範囲が \(\displaystyle\large{-\frac{\pi}{2} < \theta < \frac{\pi}{2}}\) であるため、\(\large{\tan\theta = 1}\) の解は \(\displaystyle\large{\theta=\frac{\pi}{4}}\) となります。

以上から、
最大値はなし、\(\displaystyle\large{\theta = \frac{\pi}{4}}\) のとき 最小値 \(\large{2}\)
となります。

問題(6) 三角関数の合成

以下の三角関数の最大値・最小値の問題の解き方について解説します。

問題(6)
\(\displaystyle \large{0 \leqq \theta < 2\pi }\) のとき、次の関数の最大値・最小値を求めよ。
\(\displaystyle \large{y= \sin \theta - \sqrt{3}\cos\theta}\)

【解答と解説】
問題の関数は、一次の \(\large{\sin}\) と \(\large{\cos}\) の差であるため、三角関数の合成により式を変形して解きます。

三角関数の合成は、 $$\large{a \sin \theta + b \cos \theta = r \sin (\theta + \alpha)}$$ に対して、\(\large{r=\sqrt{a^2+b^2}}\)、\(\displaystyle \large{\sin \alpha = \frac{b}{\sqrt{a^2+b^2}}}\)、\(\displaystyle \large{\cos \alpha = \frac{a}{\sqrt{a^2+b^2}}}\) を求めることで変形できます。

問題の関数に三角関数の合成を適用すると、 $$\large\sin\theta -\sqrt{3}\cos \theta = 2\sin(\theta + \alpha)$$ ここで、\(\large{\displaystyle \sin \alpha = -\frac{\sqrt{3}}{2}}\)、\(\displaystyle \large{\cos \alpha = \frac{1}{2}}\) を満たします。

すなわち、問題の関数を変形すると $$\large{y=\sin\theta -\sqrt{3}\cos \theta = 2\sin\left(\theta - \frac{\pi}{3}\right)}$$ となります。

ここで、\(\displaystyle\large{\theta- \frac{\pi}{3} = t}\) とおきます。このとき、変数 \(\large{t}\) の範囲は \(\displaystyle\large{0 \leqq \theta < 2\pi }\) から、 $$\displaystyle\large{-\frac{\pi}{3} \leqq t < \frac{5}{3}\pi}$$ となります。

問題の関数を変数\(\large{t}\) で表すと、 $$\displaystyle\large{y=2\sin t}$$ となります。\(\displaystyle\large{-\frac{\pi}{3} \leqq t < \frac{5}{3}\pi}\) の範囲において、
\(\large{y=2\sin t}\) は \(\displaystyle\large{t=\frac{\pi}{2}}\) で最大値 \(\large{2}\)、 \(\displaystyle\large{t=\frac{3}{2}\pi}\) で最小値 \(\large{-2}\) をとります。

ここで、\(\displaystyle\large{\theta-\frac{\pi}{3} = t}\) であることから
\(\displaystyle\large{\theta=\frac{5}{6}\pi}\) のとき最大値 \(\displaystyle\large{2}\)、\(\displaystyle\large{\theta=\frac{11}{6}\pi}\) のとき最小値 \(\large{-2}\)
となります。

問題(7) 二倍角の公式 と 三角関数の合成

以下の三角関数の最大値・最小値の問題の解き方について解説します。

問題(7)
\(\displaystyle \large{0 \leqq \theta \leqq \frac{\pi}{2} }\) のとき、次の関数の最大値・最小値を求めよ。
\(\displaystyle \large{y= \cos^2 \theta + 2\sin\theta \cos\theta -\sin^2 \theta}\)

【解答と解説】
問題の関数は 二倍角の公式を利用することで、一次の \(\large{\sin2\theta}\)、\(\large{\cos2\theta}\) の式に変換することができます。

二倍角の公式は、以下の式で表されます。 $$\large{\sin^2\theta = \frac{1-\cos2\theta}{2}}$$ $$\large{\cos^2\theta = \frac{1+\cos2\theta}{2}}$$ $$\large{\sin\theta \cos\theta = \frac{\sin 2\theta}{2}}$$

問題の関数を変形すると、 $$\large{y=\frac{1+\cos2\theta}{2} + \sin2\theta -\frac{1-\cos2\theta}{2} }$$ $$\large{y=\sin2\theta +\cos2\theta }$$ ここで、三角関数の合成を適用すると、 $$\large{y= \sqrt{2}\sin\left(2\theta + \frac{\pi}{4}\right)}$$ となります。

ここで、\(\displaystyle\large{2\theta+ \frac{\pi}{4} = t}\) とおきます。このとき、変数 \(\large{t}\) の範囲は \(\displaystyle\large{0 \leqq \theta \leqq \frac{\pi}{2} }\) から、 $$\displaystyle\large{0 \leqq 2\theta \leqq \pi}$$ $$\displaystyle\large{\frac{\pi}{4} \leqq 2\theta+ \frac{\pi}{4} \leqq \frac{5}{4}\pi}$$ $$\displaystyle\large{\frac{\pi}{4} \leqq t \leqq \frac{5}{4}\pi}$$ となります。

ここで、\(\displaystyle\large{\frac{\pi}{4} \leqq t \leqq \frac{5}{4}\pi}\) を満たす範囲を単位円上に描くと以下のようになります。 二倍角の公式、三角関数の合成を利用した最大値・最小値の求め方

上図より、\(\displaystyle\large{\frac{\pi}{4} \leqq t \leqq \frac{5}{4}\pi}\) の範囲において \(\large{y=\sqrt{2}\sin t}\) は
\(\displaystyle\large{t=\frac{\pi}{2}}\) で最大値 \(\large{\sqrt{2}\times1=\sqrt{2}}\)
\(\displaystyle\large{t=\frac{5}{4}\pi}\) で最小値 \(\displaystyle\large{\sqrt{2}\times\left(-\frac{\sqrt{2}}{2}\right)=-1}\) をとります。

ここで、\(\displaystyle\large{2\theta + \frac{\pi}{4} = t}\) であることから
\(\displaystyle\large{\theta=\frac{\pi}{8}}\) のとき最大値 \(\displaystyle\large{\sqrt{2}}\)、\(\displaystyle\large{\theta=\frac{\pi}{2}}\) のとき最小値 \(\large{-1}\)
となります。


Copyright (c) 光学技術の基礎用語 All Rights Reserved.